PT13.S2.Q9 - Bellaria fill in the blank/MSS

moonstars5678moonstars5678 Member
edited January 2022 in Logical Reasoning 166 karma

Really struggled with this question in terms of understanding the QS and choosing between A and D (specifically, why A isn't right). Could someone correct my understanding?

Stim:
profit --> build more rooms or improve existing
/build more rooms


/profit

My thinking: So if there's no ability to increase profits, then hotel owners are either unable to build more rooms (which stim states) and/or unable to improve existing rooms

This question is a fill in the blank, so we're looking for a most strongly supported AC.

ACs:
A: Struggled a lot with this AC - it almost seemed like a necessary assumption because sure, even if hotel owners can't build more rooms, what if their rooms are at 10% occupancy right now? They'd be able to increase profits if they upped the occupancy rate to 100%.

B: Wrong. How would we know if they are maxed out when it comes to attractive location? Stim doesn't tell us this

C: Wrong. Cost of labor? came from left field

D: Initially chose this answer and then went back to A. My thinking here originally was: Alright, so hotel owners can't build more rooms, but they have another lever. Could they do some renovations and charge the rooms higher so they can make a profit? But, this AC shows that the hotels are already maxed out when it comes to improvement so they're really just...stuck.

(but then I came back again to this question and thought, well, what if the rooms aren't actually maxed out so then I kept bouncing back and forth between A and D) :(

E: Wrong. How do we know this/what would this even do to drive towards the fact that profits aren't increasing?

help

Sign In or Register to comment.